High School Math Solving Detailed Process Thank you 20

Updated on educate 2024-04-25
10 answers
  1. Anonymous users2024-02-08

    Two +1 3; 8、-2/3; 9、an=2^(n-2)(a1=1);

    5 sub-questions: a(n+1)=a1*q n,sn=(a1-a1*q n) (1-q)=a1*(1-q n) (1-q),s2n=a1*(1-q 2n) (1-q);

    then tn=(17sn-s2n) a(n+1)=[17*a1*(1-q n)-a1*(1-q 2n)] [a1*q n*(1-q)].

    17-17q^n+q^2n)/[q^n(1-q)]=[(17/q^n)-17+q^n]/(1-q)=[(17/x)-17+x]/(1-q);…x=q^n;

    Because 1-q<0, it is clear that when (17 x)-17+x takes the minimum value, tn is maximum;

    When (17 x)=x, i.e., x= (17), the value of the above equation is the smallest, i.e., q 2n=17, and substituting q= 2 yields: 2 n=17, n 4;

    i.e., the fourth term of tn is the largest;

    15 sub-questions: (1) Let the first term of the series be a1 and the common ratio is q, then sn=a1*(1-q n) (1-q)=[a1 (1-q)]-a1 (1-q)]*q n;

    Comparing the expressions sn and y, we can see that -a1 (q-1)=-1= r; b=q;

    2)b=2=q, then y=2 x-1, sn=2 n-1=a1*(1-2 n) (1-2)=a1*(2 n-1);

    a1=1;an=q^(n-1)=2^(n-1);bn=(n+1)/[4*2^(n-1)]=(n+1)/2^(n+1);

    tn=σbn=σ[(n+1)/2^(n+1)];2*tn=σ[(n+1)/2^n];

    2tn-tn=[2/2^1-(n+1)/2^(n+1)]+1/2^n]……

    tn=1-[(n+1)/2^(n-1)]+1/2)-(1/2^n)];

    tn=(3/2)-(2n+3)/2^n;

  2. Anonymous users2024-02-07

    This is a transcendent equation that cannot be solved exactly. You can only find an approximate solution.

  3. Anonymous users2024-02-06

    x+1>0, and x+1 is not equal to 1

    Then x>-1 and x is not equal to 0

    1) If -10

    In this case, log(x+1)2 is a decreasing function and greater than 0, then x+1 log(x+1)2-1 is an increasing function, then the solution is unique.

    It is not difficult to see that x=3 meets the requirements.

    So the solution is x=3

  4. Anonymous users2024-02-05

    x, and log2 (x+1) are both increments.

    Therefore, f(x)=x+log2 (x+1)-1 is also an increasing function, and at most there is only one f(0)=-1<0

    f(1)=1+1-1=1>0

    Therefore there is a unique solution in the (0,1) interval.

    It can be solved numerically: x=

  5. Anonymous users2024-02-04

    20. (1) a(n+1)-an=2/[a(n+1)+an-1] =a(n+1)-an]*[a(n+1)+an-1]=2

    bn=(an-1/2)^2=an^2-an+1/4;

    b(n+1)=(a(n+1)-1/2)^2=a(n+1)^2-a(n+1)+1/4

    b(n+1)-bn=[a(n+1)^2-an^2]-[a(n+1)-an]

    a(n+1)-an]*[a(n+1)+an-1]

    A series is a series of equal differences with a tolerance of 2.

    2) b1=(a1-1/2)^2=(1-1/2)^2=1/4

    bn=b1+(n-1)d=1/4+(n-1)*2=2n-2+1/4=(8n-7)/4

    an≥1,∴an-1/2≥0

    an-1/2=√bn =>an=1/2+√bn=[1+√(8n-7)]/2

    3) am=k=[1+√(8m-7)]/2 =>2k-1=√(8m-7)

    2k-1)^2=8m-7 =>m=[(2k-1)^2+7]/8

    That is, as long as the value of m is m=[(2k-1) 2+7] 8, am=k

    21.(1) f(x)=alnx+x 2-12x+11 => bridge circle f'(x)=a/x+2x-12

    x=4 is the extreme point, then f'(4)=0=a/4+8-12=a/4-4 =>a=16

    2) f(x)=16lnx+x^2-12x+11 f'(x)=16 x+2x-12=2(x 2-6x+8) Yubump x=2(x-2)(x-4) x

    Define the field as x>0 and let f'(x) 0, which can be solved to be x4 or x2

    That is, the monotonic increase interval of the function f(x) is (0,2][4,+).

    Order f'(x) 0, which can be solved as 2 x 4, i.e., the monotonic reduction interval of the function f(x) is [2,4].

    3) From the monotonic interval, it can be seen that the function f(x) increases monotonically on (0,2), and obtains a maximum value at x=2 to suppress the talk f(2)=16ln2-9;

    Monotonically decreasing on [2,4], the minimum f(4)=32ln2-21 is obtained at x=4

    Monotonically increasing on [4,+.

    In order for the line y=b to have 3 intersections with y=f(x), the line needs to fall between 16ln2-9 and 32ln2-21.

    That is, the value range of b is (16LN2-9,32LN2-21).

  6. Anonymous users2024-02-03

    1.(1)bn+1-bn=(an+1-1/2)^2-(an-1/2)^2=(an+1+an-1)(an+1-an)=2

    So bn is a series of equal differences.

    2)b1=1 4 So bn=2n-7 4=(an-1 2) 2 so an=(2n-7 4) 1 2+1 2 (sorry, the third question is not very good, and the writing is a bit messy, I hope it helps you) 2(1)f'(x)=a/x+2x-12

    Because x=4 is the extreme point of the function, f'(4)=0 The solution is a=16, so f(x)=16lnx+x 2-12x+11(2)f'(x)=16 x+2x-12 (x>0)let f'(x)=0 gives x=2 or 4

    When x belongs to (0,2) and (4,positive infinity), f'(x) >0, f(x) single increment when x belongs to (2,4), f'(x) <0, f(x) single minus (3) f(2) = 16ln2-9f(4)=16ln4-21, so in order to satisfy the condition b, it belongs to (16ln4-21, 16ln2-9).

  7. Anonymous users2024-02-02

    1(3) In addition, am=k has been solved to m=k*(k-1) 2+1 k(k-1) must be even, so the solved m is a positive integer, and there is an over reply.

  8. Anonymous users2024-02-01

    (1-lgx)lgx=lg(1/100)=-2

    lgx = 2 or -1 (rounded).

    x=100 is not calculated with a pen, so there should be nothing wrong.

  9. Anonymous users2024-01-31

    Solution: Derivative of f(x)=1 x*lnx, f'(x)=-lnx+1)/(xlnx)^2

    Order f'(x)=0 gives x=1 e

    At (0,1 e) f(x) increases monotonically and decreases monotonically at (1 e,1), so that at 1 e the extreme (maximum) value is obtained. f(1/e)=e

    Look at the condition again is 2 1 x > x a

    Take the logarithm ln on both sides to obtain: ln2 1 x>lnx a, that is: ln2*1 x>a*lnx is less than zero on (0,1).

    Divide both sides by the lnx variant at the same time to get 1 x*lnxeln2

    When the extreme point is the minimum:

    f'(x)=1/x+a/x^2, f''(x)=-1/x^2-2a/x^3

    f'(x)=0, 1 x+a x 2=0, x=-a

    f(-a)=ln(-a)-a/(-a)=ln(-a)+1

    If ln(-a)+1=2, then a=-e, where x=e is in the interval [1,e], f''(e) = 1 e 2>0, i.e., there is a minima.

    When the boundary value x=1 is the minimum value of the function:

    f(1)=ln1-a=2, then a=-2

    In this case, the extreme point f(-a)=f(2)=ln2+2 2=ln2+1<2, that is, it is smaller than the boundary value, so f(1) is not the minimum value of the function.

    Hence a=-e

  10. Anonymous users2024-01-30

    f(x)=(a x -1) (a x +1) because f(-x)=(a -x -1) (a -x +1)=(1 a x -1) (1 a x +1)=(1- a x) (1+a x).

    a x -1) (a x+1) = -f(x) so f(x) is an odd function.

    f'(x)=[lna * a^x (a^x+1) -a^x-1) lna * a^x]/(a^x+1)^2

    a x lna (a x+1-a x+1) (a x+1) 2=2*a x lna (a x+1) 2 It can be seen that when a > 1, f'(x)>0, f(x) is monotonically increasing;

    a<1, f'(x)<0, f(x) is monotonically decreasing;

    a<1, f'(x)=0, f(x) is a constant function.

Related questions
12 answers2024-04-25

Let pc=b, the triangle abc becomes a, then ap= (a2-b 2), and rotate the bpc 60° counterclockwise around the point b'a, apparently p'bp=60,△bpc≌△bp'a, so bp'=bp >>>More

22 answers2024-04-25

Although quadratic functions are in junior high school, they are very important in high school, and they are the focus of the college entrance examination, and they must be examined! High school represents other functions as quadratic functions. In addition, there is the distribution of roots, which you can learn online. >>>More

9 answers2024-04-25

Solution: a1=3, an+1=2an+3

an+1+3=2(an+3), a1+3=6, the series is a proportional series with 6 as the first term and 2 as the common ratio, an+3=6 2n-1=3 2n, an=3 2n 3=3(2n-1), sn=3[(21-1)+(22-1)+(23-1)+....2n-1)]=3[ 2⎛ 1-2n1-2-n]=3(2n+1-2-n). >>>More

10 answers2024-04-25

Solution: This problem can be reduced to sinb-sinc=2sina (root number 3sinc) sinb=sin(180-a-c)=sin(a+c)sin(a+c)-sinc=sinacosc-root number 3sinasinccosasinc-sinc=-root number 3sinasincsinina is not equal to 0 >>>More

21 answers2024-04-25

Organize the equations for this straight line.

y=x+1 - This is a straight line diagonally 45 to the upper right, and the coordinates of the intersection point with the y-axis are (0,1). >>>More